Separating Variables in Differential Equations: Solving dP/dt = P - P^2

Click For Summary
The discussion focuses on solving the differential equation dP/dt = P - P^2 by separating variables. Participants identify issues with partial fraction decomposition and integration, particularly regarding sign errors in calculations. One user suggests that the correct partial fraction should be 1/P - 1/(1-P), leading to the correct integration. Another user acknowledges a mistake in their U-substitution during integration, highlighting the importance of careful calculations. The conversation emphasizes the need for accuracy in mathematical steps to arrive at the correct solution.
bdh2991
Messages
102
Reaction score
0

Homework Statement


dP/dt=P-P^2


Homework Equations





The Attempt at a Solution



I know you can separate this and after i did that and did my partial fractions i got

t + C = ln(P) + ln(1-P) but i don't know what to do from here i figure you take the e of both sides at some point but i never ended up with the right andswer please help
 
Physics news on Phys.org
bdh2991 said:

Homework Statement


dP/dt=P-P^2


Homework Equations





The Attempt at a Solution



I know you can separate this and after i did that and did my partial fractions i got

t + C = ln(P) + ln(1-P) but i don't know what to do from here i figure you take the e of both sides at some point but i never ended up with the right andswer please help

I think you have a sign error in your solution. You need to fix that and remember the formulas like ln(a)+ln(b) = ln(ab) and ln(a) - ln(b) = ln (a/b).
 
i'm not seeing where the sign error is coming from...for my partial fractions i did

A/P + B/(1-P), then getting rid of the denominators i get A(1-P) + Bp = 1

solving for A and B i get A=1, B=1

so wouldn't it be ln(P) + ln(1-P)??

the only thing i could think is that it should be A(1-P) - BP

but i don't really understand how it could come out as a subtraction
 
bdh2991 said:
i'm not seeing where the sign error is coming from...for my partial fractions i did

A/P + B/(1-P), then getting rid of the denominators i get A(1-P) + Bp = 1

solving for A and B i get A=1, B=1

so wouldn't it be ln(P) + ln(1-P)??

the only thing i could think is that it should be A(1-P) - BP

but i don't really understand how it could come out as a subtraction

Your partial fraction should be 1/P - 1/(P-1).
 
bdh2991 said:
i'm not seeing where the sign error is coming from...for my partial fractions i did

A/P + B/(1-P), then getting rid of the denominators i get A(1-P) + Bp = 1

solving for A and B i get A=1, B=1

so wouldn't it be ln(P) + ln(1-P)??

Your sign is wrong on your integration of$$
\int\frac 1 {1-P}\, dP$$
 
I see i was messing up my U-substitution, it's always the little things lol thanks for the help
 
Question: A clock's minute hand has length 4 and its hour hand has length 3. What is the distance between the tips at the moment when it is increasing most rapidly?(Putnam Exam Question) Answer: Making assumption that both the hands moves at constant angular velocities, the answer is ## \sqrt{7} .## But don't you think this assumption is somewhat doubtful and wrong?

Similar threads

  • · Replies 3 ·
Replies
3
Views
1K
  • · Replies 25 ·
Replies
25
Views
2K
Replies
2
Views
1K
Replies
11
Views
2K
  • · Replies 4 ·
Replies
4
Views
2K
  • · Replies 3 ·
Replies
3
Views
5K
Replies
3
Views
1K
  • · Replies 7 ·
Replies
7
Views
6K
  • · Replies 8 ·
Replies
8
Views
2K
  • · Replies 5 ·
Replies
5
Views
2K